Chương 4: BẤT ĐẲNG THỨC, BẤT PHƯƠNG TRÌNH

Anh Đỗ Nguyễn Thu

Cho a,b,c là các số thực dương thỏa mãn abc=1. CMR

\(\frac{1}{1+a^3+b^3}+\frac{1}{1+b^3+c^3}+\frac{1}{1+c^3+a^3}\le1\)

Nguyễn Việt Lâm
31 tháng 5 2020 lúc 21:58

Với mọi x;y dương ta có:

\(x^3+y^3=\left(x+y\right)\left(x^2+y^2-xy\right)\ge\left(x+y\right)\left(2xy-xy\right)=xy\left(x+y\right)\)

Áp dụng:

\(VT=\frac{1}{a^3+b^3+1}+\frac{1}{b^3+c^3+1}+\frac{1}{c^3+a^3+1}\)

\(VT\le\frac{1}{ab\left(a+b\right)+1}+\frac{1}{bc\left(b+c\right)+1}+\frac{1}{ca\left(c+a\right)+1}\)

\(VT\le\frac{abc}{ab\left(a+b\right)+abc}+\frac{abc}{bc\left(b+c\right)+abc}+\frac{abc}{ca\left(c+a\right)+abc}\)

\(VT\le\frac{c}{a+b+c}+\frac{a}{a+b+c}+\frac{b}{a+b+c}=1\)

Dấu "=" xảy ra khi \(a=b=c=1\)

Bình luận (0)

Các câu hỏi tương tự
lữ thị xuân nguyệt
Xem chi tiết
Tình Nguyễn Hữu
Xem chi tiết
TXT Channel Funfun
Xem chi tiết
Kim Taehyung
Xem chi tiết
Tùng Trần Sơn
Xem chi tiết
TXT Channel Funfun
Xem chi tiết
Cao Thi Thuy Duong
Xem chi tiết
dam thu a
Xem chi tiết
dam thu a
Xem chi tiết